LSAT and Law School Admissions Forum

Get expert LSAT preparation and law school admissions advice from PowerScore Test Preparation.

User avatar
 Dave Killoran
PowerScore Staff
  • PowerScore Staff
  • Posts: 5852
  • Joined: Mar 25, 2011
|
#44235
Complete Question Explanation
(The complete setup for this game can be found here: lsat/viewtopic.php?t=16650)

The correct answer choice is (B)

Because this game features eight reports filling six spaces, any time two students are eliminated from the scheduling, the remaining six students must be scheduled. This inference is applied in this question. The question stem in this question indicates that H, K, and L occupy the morning spaces, leaving only the afternoon spaces open. From the second rule, we can determine that neither O nor R can give an afternoon report. Thus, the afternoon spaces must be occupied by G, I, and N in some order. The rules and question stem indicate that G must give a report on Tuesday afternoon, and therefore N cannot give a report on Monday afternoon from the contrapositive of the third rule. Instead, N must give her report on Wednesday afternoon and I is forced to give his report on Monday afternoon. Answer choice (B) is therefore correct.
 srcline@noctrl.edu
  • Posts: 243
  • Joined: Oct 16, 2015
|
#22191
Hello

With this game I didn't seem to have trouble with the set up by I am getting tripped up on the last couple of questions.....

I am completely stumped on Question 6... I know the last rule plays a part here but I am not seeing the inference. I eliminated A B C because HI cant be on the same day, because N gives a report on W, and C is gone because it violates the stimulus given in the question. Is this correct reasoning?
I am in between question D and E

Thank you
Sarah
 Emily Haney-Caron
PowerScore Staff
  • PowerScore Staff
  • Posts: 577
  • Joined: Jan 12, 2012
|
#22192
Hi Sarah,

For question 6, if H, K, and L give the morning reports, we know the two unused variables are O and R, because they cannot go in the afternoon and there are no other morning reports left. So our afternoon variables have to be G, I, and N. G HAS to be on Tuesday afternoon. Keep in mind the rule about N, H, and I - it is critical here. With G on Tuesday, N cannot be on Monday, because that would require the HI block on Tuesday, which is impossible. N can't be Tuesday, because G is Tuesday afternoon. Therefore, N has to be on Wednesday afternoon. That leaves I for Monday afternoon.

Now let's look at the answer choices.

We know I has to be Monday; that is answer choice B. We don't even need to create hypos for the other answer choices; we know B is the one.

Get the most out of your LSAT Prep Plus subscription.

Analyze and track your performance with our Testing and Analytics Package.